passaggio matematico in una formula

Messaggioda lantis » 10/02/2009, 21:06

ciao a tutti! ragazzi non riesco a girare questa forrmula.

allora poichè $q=(T_1-T_2)/R_t$

dove $R_t$ è
Immagine

dopo vari passaggi si giunge a questa forma

Immagine

scusate se ho postato le immagini delle formule ma erano lunghissime da scrivere, spero si capisca lo stesso! cmq non riesco a capire come sia giunto alla formula finale, qualcuno può darmi una mano? grazie
lantis
Junior Member
Junior Member
 
Messaggio: 161 di 192
Iscritto il: 08/07/2007, 11:09

Messaggioda adaBTTLS » 10/02/2009, 21:18

i caratteri si leggono malissimo, ma ho l'impressione che abbia semplicemente fatto il minimo comun denominatore all'espressione di R(tot) e lo ha quindi portato al numeratore (2pi r I): di fatto num diviso frazione è num per la frazione inversa. non so se mi sono spiegata.
se qualcosa non torna, temo dovresti utilizzare le formule per essere più chiaro.
ciao.
Avatar utente
adaBTTLS
Cannot live without
Cannot live without
 
Messaggio: 2918 di 8319
Iscritto il: 14/05/2008, 18:35
Località: Abruzzo

Messaggioda lantis » 10/02/2009, 21:30

ma io vedo bene le immagini delle formule ke ho postato :( :( :(

cmq non mi è chiarissimo..avrebbe fatto il denominatore comune mettendo $2pir_1L$?
lantis
Junior Member
Junior Member
 
Messaggio: 162 di 192
Iscritto il: 08/07/2007, 11:09

Messaggioda lantis » 10/02/2009, 21:44

se prendo come denominatore comune $2pir_1L$ mi risulta:

$R_t=(h_i+lambda_i/r_1ln(r_2/r_1)+lambda_e/r_1ln(r_3/r_1)+(r_3h_e)/(r_1))/(2pir_1L)

EDIT: $((T_1-T_2)2pir_1L)/(h_i+lambda_i/r_1ln(r_2/r_1)+lambda_e/r_1ln(r_3/r_1)+(r_3h_e)/(r_1))
lantis
Junior Member
Junior Member
 
Messaggio: 163 di 192
Iscritto il: 08/07/2007, 11:09

Messaggioda lantis » 10/02/2009, 22:33

ok ragazzi ci sono arrivato :D
lantis
Junior Member
Junior Member
 
Messaggio: 164 di 192
Iscritto il: 08/07/2007, 11:09

Messaggioda adaBTTLS » 10/02/2009, 22:42

lantis ha scritto:se prendo come denominatore comune $2pir_1L$ mi risulta:

$R_t=(h_i+lambda_i/r_1ln(r_2/r_1)+lambda_e/r_1ln(r_3/r_1)+(r_3h_e)/(r_1))/(2pir_1L)

EDIT: $((T_1-T_2)2pir_1L)/(h_i+lambda_i/r_1ln(r_2/r_1)+lambda_e/r_1ln(r_3/r_1)+(r_3h_e)/(r_1))

no, da come sono riuscita a leggere nei grafici, pur non distinguendo tutti i caratteri, $R_t$ non è così. provo a correggere dalla tua formula:

$R_t=(1/h_i+r_1/lambda_i ln(r_2/r_1)+r_1/lambda_e ln(r_3/r_1)+(r_1)/(r_3h_e))/(2pir_1L)

$((T_1-T_2)2pir_1L)/(1/h_i+r_1/lambda_i ln(r_2/r_1)+r_1/lambda_e ln(r_3/r_1)+(r_1)/(r_3h_e))

spero sia chiaro. ciao.
Avatar utente
adaBTTLS
Cannot live without
Cannot live without
 
Messaggio: 2920 di 8319
Iscritto il: 14/05/2008, 18:35
Località: Abruzzo


Torna a Algebra, logica, teoria dei numeri e matematica discreta

Chi c’è in linea

Visitano il forum: Nessuno e 1 ospite